1
$\begingroup$

Problem

Suppose $f$ is a function from a complete metric space $X$ to a metric space $Y$, and suppose $Y$ has points $y_{0}$, $y_{1}$ such that the subsets $f^{-1}(y_{0})$ and $f^{-1}(y_{1})$ are both dense in $Y$.

Show that in the above situation, if $f$ maps every point of $X$ either to $y_{0}$ or to $y_{1}$ and if $f^{-1}(y_{1})$ is countable, then $f$ cannot be expressed as a pointwise limit of functions $X \rightarrow Y$ that are continuous at all points of $f^{-1}(y_{1})$; but that it can be expressed as a pointwise limit of functions $X \rightarrow Y$ that are continuous at all points of $f^{-1}(y_{0})$.

Question

It is well known by the Baire category theorem that $f$ can not be a pointwise limit of continuous functions $X \to Y$. However, I have no clue how to prove $f$ can not be a pointwise limit of functions being continuous at a dense set $f^{-1}(y_{1})$. Moreover, I think the "countable" condition is redundant for the first part since it has been proved that if $f$ is continuous on a dense set, then it is continuous on an uncountable dense set (e.g., see Theorem 1 in 1).

Also, I believe the "countable" condition is only helpful for the 2nd part as follows,

Assume $f^{-1}(y_{1}) = \{x_{1},x_{2},\ldots\}$.

Construct \begin{equation*} f_{n}(x) = \begin{cases} y_{0}, & x \in f^{-1}(y_{0}) \cup \{x_{n+1}, x_{n+2},\ldots\} \\ y_{1}, & x \in \{x_{1},x_{2},\ldots,x_{n}\} \end{cases} \end{equation*} It is easy to check $(f_{n})$ is continuous at $f^{-1}(y_{0})$.

$\endgroup$

1 Answer 1

0
+50
$\begingroup$

Suppose for a contradiction that the function $f$ is a pointwise limit of a sequence $(f_n)_{n\in\mathbb N}$ of functions from $X$ to $Y$ that are continuous at all points of $f^{-1}(y_{1})$. For each natrual $n$ let $C_n$ be the set of continuity points of the function $f_n$. It is well-known (see, for instance, Proposition 3.6 from [Kech]) that $C_n$ is a $G_\delta$ subset of $X$. Then $C=\bigcap_{n\in\mathbb N} C_n$ is a (dense) $G_\delta$ subset of $X$, and so Baire. That is the restriction $f|C$ is a pointwise limit of the sequence $(f_n|C)_{n\in\mathbb N}$ of continuous restrictions $f_n|C$, $n\in\mathbb N$.

For each natural $n$ put $C_n=\{x\in C:|f_m(x)-f(x)|\le 1/n$ for all natural $m\ge n\}$. Then $C$ is a union $\bigcup_{n\in\mathbb N} C_n$ of a nondecreasing family $\{C_n:n\in\mathbb N\}$. Since $C=\bigcup_{n\in\mathbb N} \overline{C_n}^C$ and $C$ is a Baire space, there exist $n\in\mathbb N$ and a nonempty open subset $U$ of $C$ such that $U\subset \overline{C_n}^C$, that is $C_n$ is dense in $U$. Let $d$ be the metric of the space $Y$ and $\varepsilon=d(y_0,y_1)>0$. Increasing $n$, if needed, we can ensure that $n\ge\tfrac 3\varepsilon$. Since $f^{-1}(y_1)$ is dense in $C$, there exists a point $x\in U\cap f^{-1}(y_1)$. Since the function $f_n$ is continuous at $x$, there exists an open neighborhood $V\subset U$ of $x$ such that $d(f_n(x),f_n(x'))<\tfrac{\varepsilon}{3}$ for each $x'\in V$. Since $f^{-1}(y_0)$ and $C$ are dense $G_\delta$ subsets of Baire space $X$, their intersection $f^{-1}(y_0)\cap C$ is a dense subset of $X$ and so of $C$. Pick any point $x'\in V\cap f^{-1}(y_0)$. Then by the triangle inequality $$d(f(x),f(x'))\le d(f(x),f_n(x))+d(f_n(x),f_n(x'))+(f_n(x'),f(x'))<$$ $$\frac{\varepsilon}{3}+\frac{\varepsilon}{3}+\frac{\varepsilon}{3}= \varepsilon.$$ Therefore $y_1=f(x)=f(x')=y_0$, a contradiction.

In the proof we used that the set $f^{-1}(y_0)$ is nonmeager (and dense) in $X$ to conclude that $f^{-1}(y_0)\cap C$ is dense in $C$. If we relax this condition then there is a counterexample: in fact, even in this example $f$ is a pointwise limit of functions from $X$ to $Y$ that are continuous at all points of $f^{-1}(y_{0})$.

References

[Kech] Alexander S. Kechris. Classical Descriptive Set Theory , Springer, 1995.

$\endgroup$
3
  • $\begingroup$ Thank you for your response. I was wonder, from your proof, do we really need the condition that $f$ maps $X$ either to $y_{1}$ or to $y_{0}$? It seems that $f^{-1}(y_{1})$ and $f^{-1}(y_{0})$ are dense may be enough $\endgroup$ Commented Mar 15, 2024 at 16:16
  • $\begingroup$ I still do not understand why $C_{n}$ is dense in $U$ for some $n$, could you please elaborate on it? Also, it seems that from your construction, the problem turns out to be whether $f$ can be a pointwise limit of continuous functions $C \to Y$, and $C$ is a Baire space, which is well known by the Baire category theorem. Is my statement correct? Thanks you $\endgroup$ Commented Mar 16, 2024 at 4:06
  • $\begingroup$ @hmeng I expanded the answer a bit. $\endgroup$ Commented Mar 16, 2024 at 5:50

You must log in to answer this question.

Start asking to get answers

Find the answer to your question by asking.

Ask question

Explore related questions

See similar questions with these tags.